LSAT and Law School Admissions Forum

Get expert LSAT preparation and law school admissions advice from PowerScore Test Preparation.

 mcassidy1
  • Posts: 8
  • Joined: Sep 03, 2019
|
#67858
Is this question an application of formal logic? I diagrammed it as BC -> H <-s-> OO (how an org operates)
 James Finch
PowerScore Staff
  • PowerScore Staff
  • Posts: 943
  • Joined: Sep 06, 2017
|
#67874
Hi M. Cassidy,

No, the first sentence doesn't actually tie into the conditional logic in that way. Instead, it should be seen as an argument by others ("some people believe") that the other propositions in the stimulus are effectively arguing against (although there isn't an explicit conclusion, we're being asked to draw one). So what is going on in this stimulus is that it is attempting to falsify the conditional statement that if an organization is hierarchical, then you know how it operates, implying that all hierarchical organizations operate the same way.

We're then given a conditional statement (BC :arrow: H), followed by an example of an organization that fits the sufficient condition, allowing us to infer that it also fits under the necessary condition of being hierarchical. Lastly, we learn that this particular hierarchical organization doesn't operate like the majority of bureaucratic organizations, all of which are hierarchical. So what can we infer? Only that not all hierarchical organizations function the same way, thus invalidating the original conditional relationship that some people believe in, given in the first sentence.

Hope this clears things up!

Get the most out of your LSAT Prep Plus subscription.

Analyze and track your performance with our Testing and Analytics Package.